Suppose a subspace is spanned by the set of vectors shown. Find a basis for the subspace, using the method of transforming a matrix to echelon form, where the columns of the matrix represent vectors spanning the subspace. 3 97 -21Basis = ? What is the dimension of the basis?

Answers

Answer 1

By transforming the given matrix to echelon form, we determined that the subspace spanned by the vectors [3 7] and [9 21] has a basis consisting of the vector [3 7], and the dimension of this subspace is 1.

Let's denote this matrix as A:

A = [3 9]

[7 21]

To transform this matrix to echelon form, we'll perform elementary row operations until we reach a triangular form, with leading entries (the leftmost nonzero entries) in each row strictly to the right of the leading entries of the rows above.

First, let's focus on the first column. We can perform row operations to eliminate the 7 below the leading entry 3. We achieve this by multiplying the first row by 7 and subtracting the result from the second row.

R2 = R2 - 7R1

This operation gives us a new matrix B:

B = [3 9]

[0 0]

At this point, the second column does not have a leading entry below the leading entry of the first column. Hence, we can consider the matrix B to be in echelon form.

Now, let's analyze the echelon form matrix B. The leading entries in the first column are at positions (1,1), which corresponds to the first row. Thus, we can see that the first vector [3 7] is linearly independent and will be part of our basis.

Since the second column does not have a leading entry, it does not contribute to the linear independence of the vectors. Therefore, the second vector [9 21] is a linear combination of the first vector [3 7].

To summarize, the basis for the given subspace is { [3 7] }. Since we have only one vector in the basis, the dimension of the subspace is 1.

To know more about matrix here

https://brainly.com/question/28180105

#SPJ4


Related Questions

evaluate the expression, (gof )(x), given the following functions. f(x)=x+2 and g(x)=x^(2)

Answers

The expression (gof)(x) simplifies to [tex]x^2[/tex] + 4x + 4.

To evaluate the expression (gof)(x), we need to substitute the function f(x) into g(x) and simplify the resulting expression.

f(x) = x + 2

g(x) = [tex]x^2[/tex]

Substituting f(x) into g(x), we have:

(gof)(x) = g(f(x))

Replacing f(x) with its value:

(gof)(x) = g(x + 2)

Now, substituting g(x) = [tex]x^2:[/tex]

(gof)(x) = (x + 2)^2

Expanding the square:

(gof)(x) = (x + 2)(x + 2)

(gof)(x) = x^2 + 4x + 4

Therefore, the expression (gof)(x) simplifies to:

(gof)(x) = x^2 + 4x + 4

To know more about expression refer here :

https://brainly.com/question/28170201#

#SPJ11

A United Nations report shows the mean family income for Mexican migrants to the United States is $26,450 per year. A FLOC (Farm Labor Organizing Committee) evaluation of 23 Mexican family units reveals a mean to be $37,190 with a sample standard deviation of $10,700. Does this information disagree with the United Nations report? Apply the 0.01 significance level.

(a) State the null hypothesis and the alternate hypothesis.

H0: µ = ________

H1: µ ? _________

(b) State the decision rule for .01 significance level. (Round your answers to 3 decimal places.)

Reject H0 if t is not between_______ and __________.

(c) Compute the value of the test statistic. (Round your answer to 2 decimal places.)

Value of the test statistic __________

(d) Does this information disagree with the United Nations report? Apply the 0.01 significance level.

Answers

(a) Null hypothesis (H₀): µ = $26,450

Alternate hypothesis (H1): µ ≠ $26,450

Reject H₀ if t is not between -2.807 and 2.807.

(c) Value of the test statistic 3.184.

(d) The information disagrees with the United Nations report at the 0.01 significance level since the calculated t-value falls outside the critical value range.

(a) State the null hypothesis and the alternate hypothesis:

The mean family income for Mexican migrants is $26,450 per year

H₀: µ = $26,450

The mean family income for Mexican migrants is not equal to $26,450 per year.

H₁: µ ≠ $26,450.

(b)

Reject H₀ if t is not between -2.807 and 2.807 (critical values for a two-tailed t-test with 22 degrees of freedom and a significance level of 0.01).

(c) Compute the value of the test statistic:

To compute the test statistic (t-value), we need the sample mean, the hypothesized population mean, the sample standard deviation, and the sample size.

Sample mean (X) = $37,190

Hypothesized population mean (µ) = $26,450

Sample standard deviation (s) = $10,700

Sample size (n) = 23

t-value = (X - µ) / (s / √n)

= ($37,190 - $26,450) / ($10,700 / √23)

= ($37,190 - $26,450) / ($10,700 / √23)

= $10,740 / ($10,700 / √23)

= 3.184

The calculated t-value is approximately 3.184.

d.  To determine if this information disagrees with the United Nations report, we compare the calculated t-value with the critical values for a two-tailed t-test with 22 degrees of freedom and a significance level of 0.01.

The critical values for a two-tailed t-test with a significance level of 0.01 and 22 degrees of freedom are approximately -2.807 and 2.807.

Since the calculated t-value of 3.184 falls outside the range -2.807 to 2.807, we reject the null hypothesis (H0) and conclude that there is evidence to suggest a disagreement with the United Nations report.

Therefore, based on the provided data and significance level, the information disagrees with the United Nations report.

To learn more on Statistics click:

https://brainly.com/question/30218856

#SPJ4

Given the following equation of a line x+6y=3, determine the slope of a line that is perpendicular.

Answers

The slope of the line perpendicular to the given line is 6.

Given the following equation of a line x+6y=3, we have to find the slope of a line that is perpendicular.

Let us rewrite the given equation in slope-intercept form. To do so, we need to isolate y on one side of the equation. x + 6y = 3 Subtract x from both sides.6y = -x + 3 Divide both sides by 6.y = -1/6 x + 1/2

Thus, the slope of the given line is -1/6.

To find the slope of a line that is perpendicular, we can use the formula: m1*m2 = -1 where m1 is the slope of the given line, and m2 is the slope of the perpendicular line. m1 = -1/6

Substituting this value in the above formula,-1/6 * m2 = -1m2 = 6

Thus, the slope of the line perpendicular to the given line is 6.

For more such questions on perpendicular

https://brainly.com/question/1202004

#SPJ8

in a firm with a multidivisional structure, the object is to try to achieve tight coordination between functions with emphasis on r

Answers

The statement that In a firm with a multidivisional structure, the object is to try to achieve tight coordination between functions with emphasis on R&D, production, and marketing is false.

What is multidivisional structure?

In this kind of structure, employees are divided into departments based on the types of products and/or geographic areas. For instance, General Electric has six product divisions: energy, capital, home & business solutions, healthcare, aviation, and transportation.

In contrast to a functional organization, which allows for greater efficiency by having only one department oversee all activities in a certain area, such as marketing, a multidivisional structure requires that a corporation have marketing units within each of its divisions.

It is untrue to say that the goal of a company with a multidivisional structure is to create close coordination between functions, with a focus on R&D, manufacturing, and marketing.

Learn more about multidivisional structure   at:

https://brainly.com/question/14992166

#SPJ4

complete question;

In a firm with a multidivisional structure, the object is to try to achieve tight coordination between functions with emphasis on R&D, production, and marketing. TRUE /FALSE

Which of the following is NOT a property of the linear correaton coefficient ? Choose the correct answer bolow A. The value of r measures the strength of a tinear relationshp B. The value of f is not affected by the choce of x or y C. The inear corretaton coefficent r is robust. That is, a single outier will ne D. The value of r is atways between −1 and 1 inclusive.

Answers

The correct answer is B. The value of f is not affected by the choice of x or y.

The linear correlation coefficient, denoted as r, is a measure of the strength and direction of a linear relationship between two variables. It ranges between -1 and 1, inclusive. A value close to -1 indicates a strong negative correlation, a value close to 1 indicates a strong positive correlation, and a value close to 0 indicates a weak or no correlation.

Property A is correct as the value of r indeed measures the strength of a linear relationship. Property C is also correct as the linear correlation coefficient is robust, meaning it is not greatly influenced by outliers in the data.

Property B is not true. The value of r can be affected by the choice of x or y. If we interchange the roles of x and y, the value of r will remain the same but its sign will change to reflect the new relationship. For example, if r = 0.8, then r will still be 0.8 if we switch x and y, but the direction of the relationship will be reversed.

In conclusion, the property that is NOT true for the linear correlation coefficient is B. The value of r can be affected by the choice of x or y.

To know more about correlation visit

https://brainly.com/question/30116167

#SPJ11

A projectile is thrown upward so that its distance above the ground, in feet, after t seconds is h=-11t^(2)+374t. After how many seconds does it reach its maximum height

Answers

The projectile reaches its maximum height after 17 seconds.

To find the time at which the projectile reaches its maximum height, we need to determine the vertex of the parabolic function h(t) = -11t^2 + 374t. The vertex represents the maximum point on the graph of the function.

The equation of the vertex can be found using the formula t = -b / (2a), where a and b are the coefficients of the quadratic equation in standard form (at^2 + bt + c).

In our case, the equation is h(t) = -11t^2 + 374t, so a = -11 and b = 374. Plugging these values into the formula, we get:

t = -374 / (2 * -11)

t = -374 / -22

t = 17

Therefore, the value of maximum height obtained is 17.

To know more about quadratic equation refer here:

https://brainly.com/question/29269455#

#SPJ11

Theory Question DI/HD level Using the standard 22number tutorial with unchanged code, I can see a spinning cube. I then set the following variables as shown below. GLfloat num9_lookAtX =0; GLfloat num10_lookAtY = 0; GLfloat num11_lookAtZ = -200; I ran the program, I can still see the cube. Given I am now looking at a point far past the far plane and nowhere near the cube, why can I still see it?

Answers

The cube is still visible because of depth buffering, which prioritizes the closest objects at each pixel, allowing the cube to be rendered and seen despite being outside the defined frustum.



The reason you can still see the cube despite looking at a point far past the far plane and nowhere near the cube is due to the rendering and projection techniques used in computer graphics. In OpenGL, objects are transformed and projected onto a 2D viewport for display.

The projection matrix, typically defined using functions like gluPerspective or glFrustum, sets the parameters for the clipping planes, including the near and far planes. These planes define the range of depth values that will be rendered. Objects outside this range are clipped and not displayed.However, even though your camera is positioned far beyond the cube and outside the defined frustum, the cube may still be visible due to depth buffering. Depth buffering ensures that only the closest objects at each pixel are displayed. As a result, if the cube is the closest object at certain pixels, it will still be rendered and visible, even though it is technically outside the frustum.

Therefore, The cube is still visible because of depth buffering, which prioritizes the closest objects at each pixel, allowing the cube to be rendered and seen despite being outside the defined frustum.

To learn more about matrix click here

brainly.com/question/29000721

#SPJ11

Dr. Wahl made a cup of coffee and she likes to drink it as soon as it cools to 130 ∘ F. The coffee is 194 ^0 F when she places the coffee on the counter to cool and after one minute, the coffee is 168 ∘ F. If the ambient temperature is a constant 70 ∘ F, how long until she drinks the coffee? The Mathematics behind the Model: In this problem we use Newton's Law of Cooling which states that the rate at which the temperature of the coffee is changing is proportional to the difference between the ambient temperature and the temperature of the coffee. The idea of proportionality is a common one in mathematics so we can review it here: a quantity z is (directly) proportional to x if z=kx, for some constant k. Let's fix a common notation for our work. Let T(t) be the temperature in degrees Fahrenheight of the coffee at time t in minutes. Let A represent the ambient air temperature and k be the constant of proportionality. 1. Write the differential equation that expresses Newton's Law of Cooling: the rate at which the temperature of the coffee is changing is proportional to the difference between the ambient temperature (A) and the temperature of the coffee (T). Hint: Use the symbols dT/dt,k,T and A. 2. What is the dependent variable in this differential equation? 3. What is the independent variable in this differential equation? 4. What is the value of the constant A ? Replace A in your differential equation and write the differential equations here. 5. This equation is separable. To solve it, separate (the variables T and t ) and integrate! Without using logarithms, write the function T(t) that solves the differential equation here. Your answer will include constants C _1 =e ^C (where C is the constant of integration) and k. 6. What are the values of T(0) and T(1) ? 7. Use the data points T(0) and T(1) to determine the two constants and write T(t) again here. 8. How much total time passes until she should begin drinking the coffee? Please answer in minutes and seconds to the nearest second.

Answers

The differential equation that expresses Newton's Law of Cooling is:

dT/dt = -k(T - A)

The dependent variable in this differential equation is T, representing the temperature of the coffee.

The independent variable in this differential equation is t, representing time in minutes.

The value of the constant A is 70 ∘ F.

The differential equation is:

dT/dt = -k(T - 70)

To solve the differential equation, we can separate the variables T and t and integrate both sides with respect to t:

1/(T - 70) dT = -k dt

Integrating both sides, we get:

ln|T - 70| = -kt + C_1

where C_1 is a constant of integration.

Exponentiating both sides, we get:

|T - 70| = e^C_1 * e^(-kt)

Now, since T cannot be negative, we can drop the absolute value signs, and we get:

T(t) = Ce^(-kt) + 70

where C = ±e^C_1 is another constant of integration.

We are given T(0) = 194 ∘ F and T(1) = 168 ∘ F. Plugging these values into the equation T(t) = Ce^(-kt) + 70, we get the following two equations:

194 = Ce^0 + 70   -->   C = 124

168 = 124e^(-k) + 70

Solving the second equation for k, we get:

k = ln(54/124)

Plugging in the values of C and k, we get:

T(t) = 124e^(-ln(54/124)t) + 70

Simplifying, we get:

T(t) = 54e^(-0.693t) + 70

We want to find the time it takes for the coffee to cool to 130 ∘ F. This means we need to solve for t in the equation T(t) = 130. Plugging in the values we found for C and k, we get:

130 = 54e^(-0.693t) + 70

Subtracting 70 from both sides and dividing by 54, we get:

0.3704 = e^(-0.693t)

Taking the natural logarithm of both sides, we get:

ln(0.3704) = -0.693t

Solving for t, we get:

t = ln(0.3704)/(-0.693) ≈ 1.93 minutes

Therefore, approximately 1 minute and 56 seconds pass until Dr. Wahl should begin drinking the coffee.

learn more about differential equation here

https://brainly.com/question/32645495

#SPJ11

Verify if the provided y is a solution to the corresponding ODE y=5e^αx
y=e ^2x y′ +y=0
y ′′ −y′ =0

Answers

The result is equal to zero, the provided y = e^(2x) is a solution to the ODE y'' - y' = 0.

To verify if the provided y is a solution to the given ODE, we need to substitute it into the ODE and check if the equation holds true.

y = 5e^(αx)

For the first ODE, y' + y = 0, we have:

y' = d/dx(5e^(αx)) = 5αe^(αx)

Substituting y and y' into the ODE:

y' + y = 5αe^(αx) + 5e^(αx) = 5(α + 1)e^(αx)

Since the result is not equal to zero, the provided y = 5e^(αx) is not a solution to the ODE y' + y = 0.

y = e^(2x)

For the second ODE, y'' - y' = 0, we have:

y' = d/dx(e^(2x)) = 2e^(2x)

y'' = d^2/dx^2(e^(2x)) = 4e^(2x)

Substituting y and y' into the ODE:

y'' - y' = 4e^(2x) - 2e^(2x) = 2e^(2x)

Since the result is equal to zero, the provided y = e^(2x) is a solution to the ODE y'' - y' = 0.

Learn more about  solution from

https://brainly.com/question/27894163

#SPJ11

Amber's Video Rentals wants to increase the quantity of videos that it sells by 1 percent. The price elasticity of demand for videos sold by Amber's Video Rentals is 0.2. What is the percentage price

Answers

The percentage price increase needed to achieve a 1 percent increase in quantity sold, given a price elasticity of demand of 0.2, would be 0.5 percent.

Price elasticity of demand measures the responsiveness of quantity demanded to a change in price. In this case, the price elasticity of demand is given as 0.2.

The formula for price elasticity of demand is:

Elasticity = (% change in quantity demanded) / (% change in price)

We want to find the percentage price increase needed to achieve a 1 percent increase in quantity sold. Let's denote the percentage change in quantity demanded as 1 percent and the percentage change in price as X percent.

0.2 = (1%)/(X%)

Cross-multiplying and solving for X, we get:

X% = (1%)/0.2

X% = 5%

Therefore, a 5 percent increase in price would result in a 1 percent increase in quantity sold.

To increase the quantity of videos sold by 1 percent, Amber's Video Rentals would need to increase the price by approximately 0.5 percent, given a price elasticity of demand of 0.2.

To know more about percentage follow the link:

https://brainly.com/question/29520143

#SPJ11

Calculate the cross product assuming that u×w=⟨5,−6,−1⟩ (4u+4w)×w=

Answers

The cross product assuming that is  (4u + 4w) × w = 4(u × w) + 0(4u + 4w) × w = 4(u × w) = 4⟨5, −6, −1⟩= ⟨20, −24, −4⟩

Given that u × w = ⟨5, −6, −1⟩

We are to find (4u + 4w) × w

We know that(4u + 4w) × w = 4(u + w) × w ......(i)u × w = |u| |w| sin θwhere, |u| = magnitude of vector

uw = angle between u and w

As we can see, we are not given the magnitude of either u or w, and nor are we given the angle between them.

Hence, we cannot calculate the vector product using the above formula.

However, we can use the following identity which will give us a useful result:

                                         (u + v) × w = u × w + v × w

So, we can write(4u + 4w) × w = (4u × w) + (4w × w)

Expanding, we get(4u + 4w) × w = 4(u × w) + 0(4u + 4w) × w = 4(u × w) = 4⟨5, −6, −1⟩= ⟨20, −24, −4⟩

Thus, the detailed answer is  (4u + 4w) × w = 4(u × w) + 0(4u + 4w) × w = 4(u × w) = 4⟨5, −6, −1⟩= ⟨20, −24, −4⟩

Learn more about cross product

brainly.com/question/29097076

#SPJ11

Let f (0) = 4 sin(0) sec² (0) + sec(0) tan(0). Find the anti derivative function, F (8), if F (0) = 0.

Answers

The antiderivative function F(x) is given by: F(x) = -4cos(x) - 4/3cot(x) + sec(x) + 4To find the antiderivative function F(x) given that f(0) = 4sin(0)sec^2(0) + sec(0)tan(0) and F(0) = 0, we need to integrate f(x) with respect to x.

First, let's simplify f(x) using trigonometric identities:

f(x) = 4sin(x)sec^2(x) + sec(x)tan(x)

Since sec^2(x) = 1 + tan^2(x), we can rewrite f(x) as:

f(x) = 4sin(x)(1 + tan^2(x)) + sec(x)tan(x)

    = 4sin(x) + 4sin(x)tan^2(x) + sec(x)tan(x)

Now, let's find the antiderivative of f(x) using integration techniques:

∫ f(x) dx = ∫ (4sin(x) + 4sin(x)tan^2(x) + sec(x)tan(x)) dx

We can integrate each term separately:

∫ 4sin(x) dx = -4cos(x) + C1, where C1 is the constant of integration

∫ 4sin(x)tan^2(x) dx = -4/3cot(x) + C2, where C2 is the constant of integration

∫ sec(x)tan(x) dx = sec(x) + C3, where C3 is the constant of integration

Now, we can combine these results to find the antiderivative function F(x):

F(x) = -4cos(x) - 4/3cot(x) + sec(x) + C, where C = C1 + C2 + C3 is the constant of integration

Given that F(0) = 0, we can substitute x = 0 into the expression for F(x):

F(0) = -4cos(0) - 4/3cot(0) + sec(0) + C = -4 + C = 0

From this, we find that C = 4.

Therefore, the antiderivative function F(x) is given by:

F(x) = -4cos(x) - 4/3cot(x) + sec(x) + 4

To learn more about derivative click here:

brainly.com/question/28984102?

#SPJ11

Audric drove 120km from Quezon City to San Pablo, Laguna to attend their family reunion. His average speed for the trip to San Pablo, Laguna was 10k(m)/(h) faster than on the way back to Quezon City, and as a result, his return trip took an hour

Answers

Audric's average speed for the entire trip is 125 km/h.

The speed of Audric during his trip to San Pablo, Laguna from Quezon City is 10 km/h faster than his speed on his way back to Quezon City. His return trip took an hour.

Find Audric's average speed for the entire trip.

Audric drove 120 km from Quezon City to San Pablo, Laguna to attend their family reunion.

Let's assume the speed of Audric on his way to San Pablo, Laguna was x km/h.

So, his speed on his way back to Quezon City was (x - 10) km/h.

Using the formula:

speed = distance/time

We can calculate the time Audric took to reach San Pablo, Laguna and his time to return to Quezon City.

Audric's time to reach San Pablo, Laguna = 120/xAudric's time to return to Quezon City

= 120/(x - 10)

According to the problem, his return trip took an hour,

so we have:

120/(x - 10) = 1

Now we can solve for x as follows:

120 = x - 10120 + 10

= xx = 130 km/h

Therefore, Audric's speed on his way to San Pablo, Laguna was 130 km/h, and his speed on his way back to Quezon City was (130 - 10) = 120 km/h.

Now, we can find Audric's average speed for the entire trip as follows:

Average speed = total distance / total time

Total distance = 120 km + 120 km = 240 km

Total time = 120/130 + 120/120

= 0.92 + 1 hours

= 1.92 hours

Average speed = 240/1.92

= 125 km/h

To know more about speed visit :

brainly.com/question/31756299

#SPJ11

A $2,800 loon is Paid bock with simple interest. If the omount Poid beck wo $3,388, Whot Was the simple interest?

Answers

the simple interest is $588.

To find the simple interest, we need to subtract the principal amount (initial loan) from the total amount paid back.

Simple Interest = Total Amount Paid Back - Principal Amount

In this case:

Principal Amount = $2,800

Total Amount Paid Back = $3,388

Simple Interest = $3,388 - $2,800

Simple Interest = $588

Therefore, the simple interest is $588.

Learn more about simple interest:

https://brainly.com/question/25845758

#SPJ11

Rework problem 19 from section 2.3 of your text involving
congressional committees. Assume that the committee consists of 8
Republicans and 4 Democrats. A subcommittee consisting of 5 people
is to be

Answers

The number of possible subcommittees consisting of 5 people from a committee of 8 Republicans and 4 Democrats is 1.

Based on the limited information provided, let's assume that the problem involves selecting a subcommittee consisting of 5 people from a committee consisting of 8 Republicans and 4 Democrats. We need to determine the number of different possible subcommittees that can be formed.

To solve this, we can use the concept of combinations. The number of combinations, denoted as "nCk," represents the number of ways to choose k items from a set of n items without regard to their order.

In this case, we want to calculate 5C5 since we need to select all 5 members for the subcommittee.

Using the formula for combinations, we have:

5C5 = 5! / (5!(5-5)!) = 5! / (5! * 0!) = 5! / 5! = 1

Therefore, there is only one possible subcommittee that can be formed, assuming we select all 5 members.

To learn more about “combinations” refer to the https://brainly.com/question/28065038

#SPJ11

Find (f∘g)(x) for the indicated functions and simplify. f(x)=8x+2,g(x)=7/(x+2)​

Answers

The composition (f∘g)(x) is given by (2x + 60)/(x + 2).

To find (f∘g)(x), we need to substitute the function g(x) into the function f(x) and simplify the expression.

f(x) = 8x + 2

g(x) = 7/(x + 2)

To find (f∘g)(x), we substitute g(x) into f(x):

(f∘g)(x) = f(g(x))

Substituting g(x) into f(x):

(f∘g)(x) = 8(g(x)) + 2

Replacing g(x) with its value:

(f∘g)(x) = 8(7/(x + 2)) + 2

Now, let's simplify the expression:

(f∘g)(x) = (56/(x + 2)) + 2

To simplify further, we need to obtain a common denominator:

(f∘g)(x) = (56/(x + 2)) + (2(x + 2)/(x + 2))

Combining the fractions:

(f∘g)(x) = (56 + 2(x + 2))/(x + 2)

Simplifying the numerator:

(f∘g)(x) = (56 + 2x + 4)/(x + 2)

(f∘g)(x) = (2x + 60)/(x + 2)

Therefore, the simplified expression for (f∘g)(x) is (2x + 60)/(x + 2).

To learn more about functions visit : https://brainly.com/question/11624077

#SPJ11

A company rents moving trucks out of two locations: St. Louis and Tampa. Some of their customers rent a truck in one city and return it in the other city, and the rest of their customers rent and return the truck in the same city. The company owns a total of 400 trucks. The company has seen the following trend: • About 30 percent of the trucks in St. Louis move to Tampa each week. • About 60 percent of the trucks in Tampa move to St. Louis each week. Suppose right now St. Louis has 330 trucks. How many trucks will be in each city after 1 week? [Round answers to the nearest whole number.] St. Louis: Tampa: If the vector i represents the distribution of trucks, where I1 is the number in St. Louis and 12 is the number in Tampa, find the matrix A so that Až is the distribution of trucks after 1 week. A = How many trucks will be in each city after 4 weeks? [Round answers to the nearest whole number.] St. Louis: Tampa: A brass manufacturer makes three different type of wholesale brass blocks from copper and zinc acco to the following matrix. Brass Blends Muntz metal 60 % 40 % High brass 65 % 35 % Copper Zinc Gilding metal 95 % 5% a) Make a 2 x 3 matrix B that contains the blending information in decimal form. In addition, the demand (in thousands of pounds) from Plant 1 is 10 High Brass, 3 Muntz metal, and 27 Gilding metal, and the demand from Plant 2 is is 12 High Brass, 3 Muntz metal, and 28 Gilding metal. b) Make a 3 x 2 matrix D for the demands at each plant. C) Find the matrix product to find each locations need for each type of metal. d) if the price of zinc is 50.58 per pound and the price of copper is 53.35 per pound. The total cost of Plant 1 is The total cost of plant 2 is

Answers

1. After 1 week, truck in St. Louis is 221 and in Tampa is 348.

a)  Blending matrix B: [tex]\left[\begin{array}{ccc}0.35&0.65&0\\0.4&0.6&0\\0.05&0.95&0\end{array}\right][/tex]  

b) Demand matrix D:  [tex]\left[\begin{array}{ccc}10&3&27\\12&3&28\end{array}\right][/tex]  

c) C = [tex]\left[\begin{array}{ccc}6.05&33.95&0\\6.8&36.2&0\end{array}\right][/tex]

d) The total cost of Plant 1 is $51.69 and the total cost of Plant 2 is $51.58.

Given information:

St. Louis currently has 330 trucks.About 30% of the trucks in St. Louis move to Tampa each week.About 60% of the trucks in Tampa move to St. Louis each week.

1. We can represent the distribution of trucks using a vector. Let the number of trucks in St. Louis as I1 and the number of trucks in Tampa as I2.

The change in the number of trucks in St. Louis is

= -0.3 x 330

= -99.

and, the change in the number of trucks in Tampa is

= 0.6 (400 - 330)

= 18.

Therefore, after 1 week, the number of trucks in St. Louis

= 330 - 99

= 231,

and the number of trucks in Tampa

= 330 + 18

= 348

a) Blending matrix B:

                                B = [tex]\left[\begin{array}{ccc}0.35&0.65&0\\0.4&0.6&0\\0.05&0.95&0\end{array}\right][/tex]  

b) Demand matrix D:

                              D = [tex]\left[\begin{array}{ccc}10&3&27\\12&3&28\end{array}\right][/tex]  

c) Matrix product:

To calculate the locations' needs for each type of metal, we can multiply matrix D by matrix B:

C = D x B

                    C =    [tex]\left[\begin{array}{ccc}10&3&27\\12&3&28\end{array}\right][/tex]  [tex]\left[\begin{array}{ccc}0.35&0.65&0\\0.4&0.6&0\\0.05&0.95&0\end{array}\right][/tex]  

                     C = [tex]\left[\begin{array}{ccc}6.05&33.95&0\\6.8&36.2&0\end{array}\right][/tex]

d) Total cost of Plant 1 = sum(C[0] x [50.58, 53.35])

Total cost of Plant 2 = sum(C[1] x [50.58, 53.35])

Performing the calculations will give us the total costs.

Total cost of Plant 1 = $51.69

and, Total cost of Plant 2 = (0.65 x $50.58) + (0.35 x $53.35)

                                          = $32.90 + $18.68

                                          = $51.58

Therefore, the total cost of Plant 1 is $51.69 and the total cost of Plant 2 is $51.58.

Learn more about Matrix here:

https://brainly.com/question/29132693

#SPJ4

Find the general solution of the following differential equation using the method of undetermined coefficients: d^2y/dx-5 dy/dx +6y=e^3x.

Answers

A = 1/6. So the particular solution is:

y_p = (1/6)e^(3x)

The general solution is then:

y = y_h + y_p = c1e^(2x) + c2e^(3x) + (1/6)e^(3x)

To solve this differential equation using the method of undetermined coefficients, we first find the homogeneous solution by solving the characteristic equation:

r^2 - 5r + 6 = 0

This factors as (r - 2)(r - 3) = 0, so the roots are r = 2 and r = 3. Therefore, the homogeneous solution is:

y_h = c1e^(2x) + c2e^(3x)

Next, we need to find a particular solution for the non-homogeneous term e^(3x). Since this term is an exponential function with the same exponent as one of the roots of the characteristic equation, we try a particular solution of the form:

y_p = Ae^(3x)

Taking the first and second derivatives of y_p gives:

y'_p = 3Ae^(3x)

y"_p = 9Ae^(3x)

Substituting these expressions into the original differential equation yields:

(9Ae^(3x)) - 5(3Ae^(3x)) + 6(Ae^(3x)) = e^(3x)

Simplifying this expression gives:

(9 - 15 + 6)Ae^(3x) = e^(3x)

Therefore, A = 1/6. So the particular solution is:

y_p = (1/6)e^(3x)

The general solution is then:

y = y_h + y_p = c1e^(2x) + c2e^(3x) + (1/6)e^(3x)

where c1 and c2 are constants determined from any initial conditions given.

Learn more about solution from

https://brainly.com/question/27894163

#SPJ11

The length of a niww rectangulat playing field is 8 yardn longer than triple the width It the perimeter of the rectanguiar playing finld is 376 yards. what are its dimensiotis? The wieh is yards

Answers

The rectangular playing field's dimensions are 85 yards by 26 yards, with a width of 26 yards.


Let x be the width of the rectangular playing field. According to the question, the length of a new rectangular playing field is 8 yards longer than triple the width. Therefore, the length of the rectangular playing field will be (3x + 8) yards.

The perimeter of the rectangular playing field is 376 yards. Thus, the formula for the perimeter of a rectangle is P = 2L + 2W, where P is the perimeter, L is the length, and W is the width. Substituting the values of L and W, we get:

2(3x + 8) + 2x = 376

6x + 16 + 2x = 376

8x + 16 = 376

8x = 360

x = 45

Therefore, the width of the rectangular playing field is 45 yards. And the length will be (3(45) + 8) = 143 yards. Hence, the dimensions of the rectangular playing field are 85 yards by 26 yards, with a width of 26 yards.

To know more about rectangle refer here:

https://brainly.com/question/30688709?referrer=searchResults

#SPJ11

in the quadratic equation the square of the sum of two consecutive even numbers is 324. what are the integers

Answers

Let x and x + 2 be two consecutive even numbers. Then, according to the problem, we can form a quadratic equation that represents the sum of the square of two consecutive even numbers.

This quadratic equation is shown as follows: [tex](x + x + 2)² = 324[/tex]Simplify the left-hand side of the equation as shown below. (2x + 2)² = 324Expand the left-hand side of the equation as shown below.

[tex]2² × (x² + 2x + 1) = 324[/tex]

Simplify the equation as shown below.

[tex]4(x² + 2x + 1) = 324[/tex]Simplify the equation as shown below.4x² + 8x - 320 = 0Divide the whole equation by 4 as shown below .[tex]x² + 2x - 80 = 0[/tex]Factor the quadratic expression as shown below.[tex](x + 10)(x - 8) = 0[/tex] Therefore, the two integers that satisfy the given quadratic equation are 10 and -8.

To know more about equation visit:

https://brainly.com/question/29657983

#SPJ11

Find the volume of the solid formed by rotating the region enclosed by y = e²+4, y = 0, x = 0, and x = 0.5 about the y-axis.

Answers

The volume of the solid formed by rotating the region enclosed by y = e²+4, y = 0, x = 0, and x = 0.5 about the y-axis is approximately 1.28 cubic units.

To calculate the volume, we can use the method of cylindrical shells. The height of each shell is given by the difference between the upper and lower bounds of y, which is e²+4 - 0 = e²+4. The circumference of each shell is given by 2πy, and the thickness of each shell is dx.

Integrating the volume formula

V = ∫(2πy)(dx) over the interval [0, 0.5],

we get ,

V = ∫[0,0.5] (2π(e²+4)) dx = 2π(e²+4)∫[0,0.5] dx = 2π(e²+4)(x)|[0,0.5] = 2π(e²+4)(0.5) = π(e²+4) ≈ 1.28 cubic units.

In summary, the volume of the solid formed by rotating the given region about the y-axis is approximately 1.28 cubic units. The calculation involves integrating the formula for the volume of cylindrical shells, considering the height, circumference, and thickness of each shell.

Learn more about volume here:
brainly.com/question/28058531

#SPJ11

If the correlation between amount of heating oil in gallons and housing price is - 0.86, then which one is the best one to describe the relationship between two variables?
a.Amount of heating oil in gallons and housing price are weakly negatively linearly related.
b.Amount of heating oil in gallons and housing price are weakly negatively related.
c.Amount of heating oil in gallons and housing price are highly negatively related.
d.Amount of heating oil in gallons and housing price are highly negatively linearly related.

Answers

d. Amount of heating oil in gallons and housing price are highly negatively linearly related.

The correlation coefficient (-0.86) indicates a strong negative linear relationship between the amount of heating oil in gallons and housing price. The closer the correlation coefficient is to -1 or 1, the stronger the linear relationship. In this case, the correlation coefficient of -0.86 suggests a strong negative linear relationship between the two variables.

To know more about linear visit:

brainly.com/question/31510530

#SPJ11

a motorcycle and the rider have a combined mass of 300.0kg the rider applies the brakes causing the motorcycle to accelerate at -(5.00m)/(s^(2)) whats the magnitude of the net force on the motorcycle

Answers

If a motorcycle and the rider have a combined mass of 300.0kg the rider applies the brakes causing the motorcycle to accelerate at -(5.00m)/(s²), then the magnitude of the net force on the motorcycle is 1500 N.

To find the net force, follow these steps:

The formula for net force is Fnet = ma, where Fnet is the net force acting on the object, m is the mass of the object and a is the acceleration of the object.The mass of the motorcycle and the rider is 300.0 kg. The acceleration of the motorcycle is -5.00 m/s2 (negative sign because the motorcycle is decelerating due to the brakes being applied). So, Fnet = 300.0 kg x (-5.00 m/s2) ⇒Fnet = -1500 N. The magnitude of the net force is 1500 N (the negative sign indicates that the force is acting in the opposite direction of the motion of the motorcycle).

Learn more about net force:

brainly.com/question/12970081

#SPJ11

Factor the following function by finding all rational and other zeros first: P(x)=x^(3)+2x^(2)+x+2.

Answers

The complete factorization of the function P(x) is [tex]P(x) = (x + 1)(x - [-1 + i*\sqrt{ (7)/ 2} (x - [-1 - i*\sqrt{(7)] / 2}.[/tex]

The function given to us is: P(x) = x³ + 2x² + x + 2

To find all the rational and other zeros of the given function, we can use the rational root theorem. According to the rational root theorem, if a polynomial function has a rational zero, then it must be of the form: p/q where p is a factor of the constant term of the function and q is a factor of the leading coefficient of the function.

Here, the constant term is 2 and the leading coefficient is 1, so the possible rational roots of the function P(x) are: ±1, ±2.

Next, we can test these possible rational roots using synthetic division:

Let's start with the root x = -1, we have the following synthetic division:

x  |  1  2  1  2-1  |___|_______|_______|______|1   1   2  |  0

Since we get a zero remainder, x = -1 is a root of the function P(x).Using the factor theorem, we can write:

P(x) = (x + 1)(x² + x + 2)

Now, we need to find the roots of the quadratic factor x² + x + 2. Since there are no real roots of this quadratic, we can use the quadratic formula to find the complex roots:

x = [-b ± sqrt(b² - 4ac)] / 2a

Here, a = 1, b = 1, c = 2, so we have:

[tex]x = [-1 ± sqrt(1 - 4(1)(2))] / 2[/tex]

[tex]= [-1 ± sqrt(-7)] / 2[/tex]

[tex]= [-1 ± i*sqrt(7)] / 2[/tex]

To know more about the function, visit:

https://brainly.com/question/29633660

#SPJ11

Using your calculator matrix mode, solve the system of equations using the inverse of the coefficient matrix. Show all matrices. Keep three decimal places in your inverse matrix. x−2y=−33x+y=2​

Answers

The solution of the given system of equations is [tex]$\left(\begin{matrix}-1 \\ -\frac{17}{7}\end{matrix}\right)$ .[/tex]

Given system of equations: x - 2y = -3x + y = 2We can represent it as a matrix:[tex]$$\left(\begin{matrix}1 & -2 \\ 3 & 1\end{matrix}\right)\left(\begin{matrix}x \\ y\end{matrix}\right) = \left(\begin{matrix}-3 \\ 2\end{matrix}\right)$$[/tex].Let's name this matrix A. Then the system can be written as:[tex]$$A\vec{x} = \vec{b}$$[/tex] We need to find inverse of matrix A:[tex]$$A^{-1} = \frac{1}{\det(A)}\left(\begin{matrix}a_{22} & -a_{12} \\ -a_{21} & a_{11}\end{matrix}\right)$$where $a_{ij}$[/tex]are the elements of matrix A. Let's calculate the determinant of A:[tex]$$\det(A) = \begin{vmatrix}1 & -2 \\ 3 & 1\end{vmatrix} = (1)(1) - (-2)(3) = 7$$[/tex]

Now, let's calculate the inverse of A:[tex]$$A^{-1} = \frac{1}{7}\left(\begin{matrix}1 & 2 \\ -3 & 1\end{matrix}\right)$$[/tex]We can solve the system by multiplying both sides by [tex]$A^{-1}$:$$A^{-1}A\vec{x} = A^{-1}\vec{b}$$$$\vec{x} = A^{-1}\vec{b}$$[/tex]Substituting the values, we get:[tex]$$\vec{x} = \frac{1}{7}\left(\begin{matrix}1 & 2 \\ -3 & 1\end{matrix}\right)\left(\begin{matrix}-3 \\ 2\end{matrix}\right)$$$$\vec{x} = \frac{1}{7}\left(\begin{matrix}-7 \\ -17\end{matrix}\right)$$$$\vec{x} = \left(\begin{matrix}-1 \\ -\frac{17}{7}\end{matrix}\right)$$[/tex]

Let's learn more about matrix:

https://brainly.com/question/31397722

#SPJ11

i) Are the following equalities generally valid? A ∪ (B \ C) = (A ∪ B) \ (A ∪ C)
A ∩ (B \ C) = (A ∩ B) \ (A ∩ C)
Give a counterexample or prove the argument
ii) Give an example of a set A containing at least one element that fulfills the condition
if x ∈ A so {x} ∈ A

Answers

1.  The equalities are not generally valid.

2. 0 is an element of A, and {0} is also an element of A since it is a singleton set containing 0.

i) The equalities A ∪ (B \ C) = (A ∪ B) \ (A ∪ C) and A ∩ (B \ C) = (A ∩ B) \ (A ∩ C) are not generally valid.

Counterexample for A ∪ (B \ C) = (A ∪ B) \ (A ∪ C):

Let A = {1, 2}, B = {2, 3}, and C = {1, 3}.

A ∪ (B \ C) = {1, 2} ∪ {2} = {1, 2}

(A ∪ B) \ (A ∪ C) = ({1, 2} ∪ {2, 3}) \ ({1, 2} ∪ {1, 3}) = {1, 2, 3} \ {1, 2} = {3}

Since {1, 2} is not equal to {3}, the equality A ∪ (B \ C) = (A ∪ B) \ (A ∪ C) does not hold in this case.

Counterexample for A ∩ (B \ C) = (A ∩ B) \ (A ∩ C):

Let A = {1, 2}, B = {2, 3}, and C = {1, 3}.

A ∩ (B \ C) = {1, 2} ∩ {2} = {2}

(A ∩ B) \ (A ∩ C) = ({1, 2} ∩ {2, 3}) \ ({1, 2} ∩ {1, 3}) = {2} \ {1, 2} = {}

Since {2} is not equal to {}, the equality A ∩ (B \ C) = (A ∩ B) \ (A ∩ C) does not hold in this case.

Therefore, the equalities are not generally valid.

ii) An example of a set A containing at least one element that fulfills the condition if x ∈ A, then {x} ∈ A is:

A = {0, {0}}

In this case, 0 is an element of A, and {0} is also an element of A since it is a singleton set containing 0.

Learn more about Elements from

https://brainly.com/question/25916838

#SPJ11

HELP PLEASE WILL MARK BRAINLIEST. Leo walk 7km outh then 12km eat. How far i he from the tarting point

Answers

Leo is approximately 13.928 km away from the starting point.

Given that Leo walked 7 km south and then 12 km east, we need to determine the distance from the starting point,

To determine the distance from the starting point, we can use the Pythagorean theorem, which states that in a right triangle, the square of the hypotenuse (the side opposite the right angle) is equal to the sum of the squares of the other two sides.

In this case, the distance Leo walked south forms one side of a right triangle, and the distance he walked east forms the other side. The distance from the starting point will be the length of the hypotenuse.

Using the Pythagorean theorem, we can calculate the distance from the starting point as follows:

Distance² = (7 km)² + (12 km)²

Distance² = 49 km² + 144 km²

Distance² = 193 km²

Taking the square root of both sides gives us:

Distance = √(193)

Distance ≈ 13.928 km

Therefore, Leo is approximately 13.928 km away from the starting point.

Learn more about Pythagorean theorem click;

https://brainly.com/question/14930619

#SPJ4

Complete question =

Leo walk 7km south then 12km east. How far is he from the starting point?

Suppose that a random sample of 18 adults has a mean score of 64 on a standardized personality test, with a standard deviation of 4. (A higher score indicates a more personable participant.) If we assume that scores on this test are normally distributed, find a 95% confidence interval for the mean score of all takers of this test. Give the lower limit and upper limit of the 95% confidence interval.
Carry your Intermediate computations to at least three decimal places. Round your answers to one decimal place. (If necessary, consult a list of formulas.)
Lower limit:
Upper limit:

Answers

To find the 95% confidence interval for the mean score of all takers of the test, we can use the formula:

Confidence Interval = sample mean ± (critical value * standard error)

First, we need to calculate the critical value. Since the sample size is 18 and we want a 95% confidence level, we look up the critical value for a 95% confidence level and 17 degrees of freedom (n-1) in the t-distribution table. The critical value is approximately 2.110.

Next, we calculate the standard error, which is the standard deviation of the sample divided by the square root of the sample size:

Standard Error = standard deviation / sqrt(sample size)

              = 4 / sqrt(18)

              ≈ 0.943

Now we can calculate the confidence interval:

Confidence Interval = sample mean ± (critical value * standard error)

                   = 64 ± (2.110 * 0.943)

                   ≈ 64 ± 1.988

                   ≈ (62.0, 66.0)

Therefore, the 95% confidence interval for the mean score of all takers of the test is approximately (62.0, 66.0). The lower limit is 62.0 and the upper limit is 66.0.

Learn more about confidence interval here:

https://brainly.com/question/32546207

#SPJ11

I. Both paira of opposite sides are congruent II. Any two consecutive angles are supplementary III. Both pairs of opposite aides are parallel IV. Both pairs of opposite angles are confruent

Answers

The set of conditions that describe a parallelogram are as follows:

Both pairs of opposite sides are congruent.

Both pairs of opposite sides are parallel.

Both pairs of opposite angles are congruent.

Any two consecutive angles are supplementary.

The above-described properties are used to define a parallelogram. A parallelogram is defined as a quadrilateral with both pairs of opposite sides parallel and congruent.The opposite sides of a parallelogram are congruent. All sides of the parallelogram are parallel to each other. The opposite angles are congruent and are equal in size. The consecutive angles are supplementary, meaning they add up to 180 degrees. The diagonal of a parallelogram bisects each other.

Parallelograms come in a variety of shapes and sizes. The properties of a parallelogram will remain the same regardless of its size or shape. All parallelograms are quadrilaterals, and they are categorized as such because they have four sides.

The parallelogram has several intriguing properties. Its properties, including its opposite sides being parallel and congruent, make it unique.The sum of the interior angles of a parallelogram is always 360 degrees. Additionally, the area of the parallelogram is equivalent to the product of its base and height.


To know more about parallelogram click here:

https://brainly.com/question/32441125

#SPJ11

the population of the town of chestnut hulls increased at a steady rate from 19,800 in 2001 to 21,400 in 2010. on average which towns population grew faster? what was the average rate of growth for the fastest growing town?

Answers

Chestnut Hills grew faster with an average growth rate of 1,600 people per decade, while the growth rate for Walnut Park is unknown based on the given information.

The correct answer is option D.

Based on the information provided, we can calculate the average rate of growth for each town and determine which town grew faster.

For Chestnut Hills:

Population in 2001 = 19,800

Population in 2010 = 21,400

Number of years = 2010 - 2001 = 9

Change in population = 21,400 - 19,800 = 1,600

Average rate of growth = Change in population / Number of years = 1,600 / 9 = 177.78 (rounded to the nearest whole number)

For Walnut Park, the graph does not provide specific population values for each year, so we cannot calculate the exact rate of growth. However, based on the given options, we can conclude that the average rate of growth for Walnut Park must be less than 1,600 people per decade, as Chestnut Hills had a growth rate of 1,600 people per decade.

Therefore, the correct answer is: D. Chestnut Hills grew faster. It grew by 1,600 people per decade.

For more such information on: growth rate

https://brainly.com/question/30611694

#SPJ8

Other Questions
Is federal tax the same in every state? Which of the following statements is True:(A) A European call option gives the holder the right to buy the underlying at a predefined price and at any time in the future.(B) A European call option gives the holder the obligation to buy the underlying at a predefined price and at any time in the future.(C) A European call option gives the holder the right to buy the underlying at a predefined price and a predefined time in the future.(D) A European call option gives the holder the obligation to buy the underlying at a predefined price and a predefined time in the future.(E) A European call option gives the holder the right to buy the underlying at any price and a predefined time in the future. Solve the following problem using the northwest corner algorithm.a=( 252550) b=( 15203035) C= 1089523674768 in a(n) ________ situation, the buyer wants to revise product specifications, prices, terms, or suppliers. who is the leader of the house of representatives; who is the leader of the senate; what is the leader of the senate called; what does the speaker of the house do; who are the leaders of the house and senate; duties of a speaker; house minority leader definition; what is the leader of the house of representatives called Solve the Second Order Equation with Complex Roots: 4y^'' + 9y^'= 0 on your resume, what is the correct way to list your psychology degree from ucf? Write a C++ program to initialize two float variables by using new operator, print the smaller number and then delete all variables using delete operator. Use pointers and references. According to the new classical theory, a exist200 billion increase in government expenditures financed by a exist200 billion increase in the budget deficit will (more than one answer is correct): a. have little or no effect on real or nominal interest rates. b. cause real output to expand. c. exert little impact on real output because higher real interest rates will crowd out private spending. d. have, no or limited effects on interest rates or real output. e. be largely offset by a reduction in private spending because individuals will anticipate higher future taxes. Which of the following statements are true and which are false? Justify your answers!(a) Let the joint density function of two random variables X and Y be given byfx.r (x, y), x 0, y x.Then X and Y are independent if fx,y can be factorised as fxr(x, y) = g(x)h (y)where g is a function of x only and h is a function of y only.(b) Assume that X and Y are two continuous random variables. If fxy (xy) = 0 for all values of x and y then X and Y are independent.(c) Assume that X and Y are two continuous random variables. If fxr (xy) = fx (y) for all values of y then X and Y are independent. Fill in the blank: In a situation where a friend or family member turns to you for comfort, the best listening style to apply would be ______ A set-associative cache consists of 64 lines, or slots, divided into four-line sets. Main memory contains 4 K blocks of 128 words each. Show the format of main memory addresses. 3. A two-way set-associative cache has lines of 16 bytes and a total size of 8kB. The 64MB main memory is byte addressable. Show the format of main memory addresses. eighty five percent of the first year students at a business school are female, while 15 % are male. school records indicates that 70% of female first year students will graduate in 3 years with a business degree, while 90% of male first year students will graduate in 3 years with a business degree. a first year student is chosen at random, the p (student will graduate) is: yvette was recently hired as a restaurant manager for mcdonald's and must attend classes at hamburger university. this is an example of ________. A) off-the-job trainingB) on-the-job trainingC) programmed learningD) online training E) Gamification Kemani WalkerLaw of SinesJun 15, 9:29:00 PM?In ATUV, t = 820 inches, m/U=132 and m2V=25. Find the length of u, to thenearest inch.Answer: u =Submit Answer which java statement allows you to use classes in other packages Behavior analytic assessments are used to identify specific target behaviors, collect baseline information, andA. diagnose specific conditionsB. modify diagnostic impressionsC. monitor treatment outcomes: Correct! If you think implied volatility is very high on a given stock and wish to trade options to profit from this mispricing, what should you do? Name a binary event when you can benefit from this type of trading. Consolidation Entry S credits the investment in subsidiary account in order toa. remove the beginning of the year book value component of the investment accountb. completely eliminate the investment accountc. allocate goodwill acquired in the business combination Ken Burrows has deposited $10,000 in an investment accountpaying 8% annual interest. How many years will it take Kensinvestment to grow to $25,000?